Asymptotic behaviour of recurrence












3















In DNA chain, there are four types of bases: A, C, G, T. Let $g(n)$ be the number of configurations of a DNA chain of length $n$, in which the sequences TT and TG never appear. Write a recurrence for $g(n)$. Determine the asymptotic behavior of the recurrence. Also, prove whether this rate of growth is $o(n^{ n^{1/2}}), Theta(n^{ n^{1/2}}),$ or $Omega(n^{ n^{1/2}})$.




My answer:
$g(n)=A(n)+C(n)+G(n)+T(n)$



$g(n)=g(n-1)+g(n-1)+g(n-1)+A(n-1)+C(n-1)$



$g(n)=3g(n-1)+2g(n-2)$



After solving this recurrence, we get



$$begin{align}
g(n) &= (frac{(4)(17)^{1/2}+16}{(3)(17)^{1/2}+17})(frac{3+(17)^{1/2})}{2})^n \
&+(frac{(17)^{1/2}-5)}{2(17)^{1/2}})(frac{3-(17)^{1/2}}{2})^n
end{align}$$



Therefore when $n$ is large, $(frac{3-(17)^{1/2})}{2})^n$ become smaller and smaller
So we can write $$g(n) = Theta((frac{3+(17)^{1/2})}{2})^n).$$



(Do I determine the asymptotic behavior of the recurrence correctly?)



But I do not know what is the rate of growth, if we take the limit of $frac{g(n)}{n^{ n^{1/2}}}$,where n tend to infinity, it seems impossible to calculate it. How could I determine the rate of growth?










share|cite|improve this question
























  • Write, for example, $Theta$ for $Theta$; this is what's known as MathJax, a type of $LaTeX$.
    – Shaun
    Nov 26 at 15:56












  • Use $frac{a}{b}$ for $frac{a}{b}$.
    – Shaun
    Nov 26 at 15:58










  • Please edit the question.
    – Shaun
    Nov 26 at 15:59










  • The exercise is strange because it has a much simpler solution. Namely, note that the total number of configurations is $4^n$ hence $g(n)leqslant4^n$. Since $4^n=o(n^{sqrt n})$, you are done.
    – Did
    Dec 12 at 9:18


















3















In DNA chain, there are four types of bases: A, C, G, T. Let $g(n)$ be the number of configurations of a DNA chain of length $n$, in which the sequences TT and TG never appear. Write a recurrence for $g(n)$. Determine the asymptotic behavior of the recurrence. Also, prove whether this rate of growth is $o(n^{ n^{1/2}}), Theta(n^{ n^{1/2}}),$ or $Omega(n^{ n^{1/2}})$.




My answer:
$g(n)=A(n)+C(n)+G(n)+T(n)$



$g(n)=g(n-1)+g(n-1)+g(n-1)+A(n-1)+C(n-1)$



$g(n)=3g(n-1)+2g(n-2)$



After solving this recurrence, we get



$$begin{align}
g(n) &= (frac{(4)(17)^{1/2}+16}{(3)(17)^{1/2}+17})(frac{3+(17)^{1/2})}{2})^n \
&+(frac{(17)^{1/2}-5)}{2(17)^{1/2}})(frac{3-(17)^{1/2}}{2})^n
end{align}$$



Therefore when $n$ is large, $(frac{3-(17)^{1/2})}{2})^n$ become smaller and smaller
So we can write $$g(n) = Theta((frac{3+(17)^{1/2})}{2})^n).$$



(Do I determine the asymptotic behavior of the recurrence correctly?)



But I do not know what is the rate of growth, if we take the limit of $frac{g(n)}{n^{ n^{1/2}}}$,where n tend to infinity, it seems impossible to calculate it. How could I determine the rate of growth?










share|cite|improve this question
























  • Write, for example, $Theta$ for $Theta$; this is what's known as MathJax, a type of $LaTeX$.
    – Shaun
    Nov 26 at 15:56












  • Use $frac{a}{b}$ for $frac{a}{b}$.
    – Shaun
    Nov 26 at 15:58










  • Please edit the question.
    – Shaun
    Nov 26 at 15:59










  • The exercise is strange because it has a much simpler solution. Namely, note that the total number of configurations is $4^n$ hence $g(n)leqslant4^n$. Since $4^n=o(n^{sqrt n})$, you are done.
    – Did
    Dec 12 at 9:18
















3












3








3


1






In DNA chain, there are four types of bases: A, C, G, T. Let $g(n)$ be the number of configurations of a DNA chain of length $n$, in which the sequences TT and TG never appear. Write a recurrence for $g(n)$. Determine the asymptotic behavior of the recurrence. Also, prove whether this rate of growth is $o(n^{ n^{1/2}}), Theta(n^{ n^{1/2}}),$ or $Omega(n^{ n^{1/2}})$.




My answer:
$g(n)=A(n)+C(n)+G(n)+T(n)$



$g(n)=g(n-1)+g(n-1)+g(n-1)+A(n-1)+C(n-1)$



$g(n)=3g(n-1)+2g(n-2)$



After solving this recurrence, we get



$$begin{align}
g(n) &= (frac{(4)(17)^{1/2}+16}{(3)(17)^{1/2}+17})(frac{3+(17)^{1/2})}{2})^n \
&+(frac{(17)^{1/2}-5)}{2(17)^{1/2}})(frac{3-(17)^{1/2}}{2})^n
end{align}$$



Therefore when $n$ is large, $(frac{3-(17)^{1/2})}{2})^n$ become smaller and smaller
So we can write $$g(n) = Theta((frac{3+(17)^{1/2})}{2})^n).$$



(Do I determine the asymptotic behavior of the recurrence correctly?)



But I do not know what is the rate of growth, if we take the limit of $frac{g(n)}{n^{ n^{1/2}}}$,where n tend to infinity, it seems impossible to calculate it. How could I determine the rate of growth?










share|cite|improve this question
















In DNA chain, there are four types of bases: A, C, G, T. Let $g(n)$ be the number of configurations of a DNA chain of length $n$, in which the sequences TT and TG never appear. Write a recurrence for $g(n)$. Determine the asymptotic behavior of the recurrence. Also, prove whether this rate of growth is $o(n^{ n^{1/2}}), Theta(n^{ n^{1/2}}),$ or $Omega(n^{ n^{1/2}})$.




My answer:
$g(n)=A(n)+C(n)+G(n)+T(n)$



$g(n)=g(n-1)+g(n-1)+g(n-1)+A(n-1)+C(n-1)$



$g(n)=3g(n-1)+2g(n-2)$



After solving this recurrence, we get



$$begin{align}
g(n) &= (frac{(4)(17)^{1/2}+16}{(3)(17)^{1/2}+17})(frac{3+(17)^{1/2})}{2})^n \
&+(frac{(17)^{1/2}-5)}{2(17)^{1/2}})(frac{3-(17)^{1/2}}{2})^n
end{align}$$



Therefore when $n$ is large, $(frac{3-(17)^{1/2})}{2})^n$ become smaller and smaller
So we can write $$g(n) = Theta((frac{3+(17)^{1/2})}{2})^n).$$



(Do I determine the asymptotic behavior of the recurrence correctly?)



But I do not know what is the rate of growth, if we take the limit of $frac{g(n)}{n^{ n^{1/2}}}$,where n tend to infinity, it seems impossible to calculate it. How could I determine the rate of growth?







discrete-mathematics asymptotics biology






share|cite|improve this question















share|cite|improve this question













share|cite|improve this question




share|cite|improve this question








edited Nov 26 at 16:06

























asked Nov 26 at 15:51









user614642

278




278












  • Write, for example, $Theta$ for $Theta$; this is what's known as MathJax, a type of $LaTeX$.
    – Shaun
    Nov 26 at 15:56












  • Use $frac{a}{b}$ for $frac{a}{b}$.
    – Shaun
    Nov 26 at 15:58










  • Please edit the question.
    – Shaun
    Nov 26 at 15:59










  • The exercise is strange because it has a much simpler solution. Namely, note that the total number of configurations is $4^n$ hence $g(n)leqslant4^n$. Since $4^n=o(n^{sqrt n})$, you are done.
    – Did
    Dec 12 at 9:18




















  • Write, for example, $Theta$ for $Theta$; this is what's known as MathJax, a type of $LaTeX$.
    – Shaun
    Nov 26 at 15:56












  • Use $frac{a}{b}$ for $frac{a}{b}$.
    – Shaun
    Nov 26 at 15:58










  • Please edit the question.
    – Shaun
    Nov 26 at 15:59










  • The exercise is strange because it has a much simpler solution. Namely, note that the total number of configurations is $4^n$ hence $g(n)leqslant4^n$. Since $4^n=o(n^{sqrt n})$, you are done.
    – Did
    Dec 12 at 9:18


















Write, for example, $Theta$ for $Theta$; this is what's known as MathJax, a type of $LaTeX$.
– Shaun
Nov 26 at 15:56






Write, for example, $Theta$ for $Theta$; this is what's known as MathJax, a type of $LaTeX$.
– Shaun
Nov 26 at 15:56














Use $frac{a}{b}$ for $frac{a}{b}$.
– Shaun
Nov 26 at 15:58




Use $frac{a}{b}$ for $frac{a}{b}$.
– Shaun
Nov 26 at 15:58












Please edit the question.
– Shaun
Nov 26 at 15:59




Please edit the question.
– Shaun
Nov 26 at 15:59












The exercise is strange because it has a much simpler solution. Namely, note that the total number of configurations is $4^n$ hence $g(n)leqslant4^n$. Since $4^n=o(n^{sqrt n})$, you are done.
– Did
Dec 12 at 9:18






The exercise is strange because it has a much simpler solution. Namely, note that the total number of configurations is $4^n$ hence $g(n)leqslant4^n$. Since $4^n=o(n^{sqrt n})$, you are done.
– Did
Dec 12 at 9:18












1 Answer
1






active

oldest

votes


















1














You solved the recurrence and found its asymptotic behavior correctly. The growth is $c^n$, where $c=frac12(3+sqrt{17})$, and you are asked how this compares with $n^{n^{1/2}}$. The latter is faster: to see this, take logs of both, and note that
$$
(log c)n=oBig(n^{1/2}log nBig)
$$






share|cite|improve this answer





















  • Thank you very much
    – user614642
    Nov 27 at 3:53











Your Answer





StackExchange.ifUsing("editor", function () {
return StackExchange.using("mathjaxEditing", function () {
StackExchange.MarkdownEditor.creationCallbacks.add(function (editor, postfix) {
StackExchange.mathjaxEditing.prepareWmdForMathJax(editor, postfix, [["$", "$"], ["\\(","\\)"]]);
});
});
}, "mathjax-editing");

StackExchange.ready(function() {
var channelOptions = {
tags: "".split(" "),
id: "69"
};
initTagRenderer("".split(" "), "".split(" "), channelOptions);

StackExchange.using("externalEditor", function() {
// Have to fire editor after snippets, if snippets enabled
if (StackExchange.settings.snippets.snippetsEnabled) {
StackExchange.using("snippets", function() {
createEditor();
});
}
else {
createEditor();
}
});

function createEditor() {
StackExchange.prepareEditor({
heartbeatType: 'answer',
autoActivateHeartbeat: false,
convertImagesToLinks: true,
noModals: true,
showLowRepImageUploadWarning: true,
reputationToPostImages: 10,
bindNavPrevention: true,
postfix: "",
imageUploader: {
brandingHtml: "Powered by u003ca class="icon-imgur-white" href="https://imgur.com/"u003eu003c/au003e",
contentPolicyHtml: "User contributions licensed under u003ca href="https://creativecommons.org/licenses/by-sa/3.0/"u003ecc by-sa 3.0 with attribution requiredu003c/au003e u003ca href="https://stackoverflow.com/legal/content-policy"u003e(content policy)u003c/au003e",
allowUrls: true
},
noCode: true, onDemand: true,
discardSelector: ".discard-answer"
,immediatelyShowMarkdownHelp:true
});


}
});














draft saved

draft discarded


















StackExchange.ready(
function () {
StackExchange.openid.initPostLogin('.new-post-login', 'https%3a%2f%2fmath.stackexchange.com%2fquestions%2f3014490%2fasymptotic-behaviour-of-recurrence%23new-answer', 'question_page');
}
);

Post as a guest















Required, but never shown

























1 Answer
1






active

oldest

votes








1 Answer
1






active

oldest

votes









active

oldest

votes






active

oldest

votes









1














You solved the recurrence and found its asymptotic behavior correctly. The growth is $c^n$, where $c=frac12(3+sqrt{17})$, and you are asked how this compares with $n^{n^{1/2}}$. The latter is faster: to see this, take logs of both, and note that
$$
(log c)n=oBig(n^{1/2}log nBig)
$$






share|cite|improve this answer





















  • Thank you very much
    – user614642
    Nov 27 at 3:53
















1














You solved the recurrence and found its asymptotic behavior correctly. The growth is $c^n$, where $c=frac12(3+sqrt{17})$, and you are asked how this compares with $n^{n^{1/2}}$. The latter is faster: to see this, take logs of both, and note that
$$
(log c)n=oBig(n^{1/2}log nBig)
$$






share|cite|improve this answer





















  • Thank you very much
    – user614642
    Nov 27 at 3:53














1












1








1






You solved the recurrence and found its asymptotic behavior correctly. The growth is $c^n$, where $c=frac12(3+sqrt{17})$, and you are asked how this compares with $n^{n^{1/2}}$. The latter is faster: to see this, take logs of both, and note that
$$
(log c)n=oBig(n^{1/2}log nBig)
$$






share|cite|improve this answer












You solved the recurrence and found its asymptotic behavior correctly. The growth is $c^n$, where $c=frac12(3+sqrt{17})$, and you are asked how this compares with $n^{n^{1/2}}$. The latter is faster: to see this, take logs of both, and note that
$$
(log c)n=oBig(n^{1/2}log nBig)
$$







share|cite|improve this answer












share|cite|improve this answer



share|cite|improve this answer










answered Nov 26 at 16:45









Mike Earnest

20.2k11950




20.2k11950












  • Thank you very much
    – user614642
    Nov 27 at 3:53


















  • Thank you very much
    – user614642
    Nov 27 at 3:53
















Thank you very much
– user614642
Nov 27 at 3:53




Thank you very much
– user614642
Nov 27 at 3:53


















draft saved

draft discarded




















































Thanks for contributing an answer to Mathematics Stack Exchange!


  • Please be sure to answer the question. Provide details and share your research!

But avoid



  • Asking for help, clarification, or responding to other answers.

  • Making statements based on opinion; back them up with references or personal experience.


Use MathJax to format equations. MathJax reference.


To learn more, see our tips on writing great answers.





Some of your past answers have not been well-received, and you're in danger of being blocked from answering.


Please pay close attention to the following guidance:


  • Please be sure to answer the question. Provide details and share your research!

But avoid



  • Asking for help, clarification, or responding to other answers.

  • Making statements based on opinion; back them up with references or personal experience.


To learn more, see our tips on writing great answers.




draft saved


draft discarded














StackExchange.ready(
function () {
StackExchange.openid.initPostLogin('.new-post-login', 'https%3a%2f%2fmath.stackexchange.com%2fquestions%2f3014490%2fasymptotic-behaviour-of-recurrence%23new-answer', 'question_page');
}
);

Post as a guest















Required, but never shown





















































Required, but never shown














Required, but never shown












Required, but never shown







Required, but never shown

































Required, but never shown














Required, but never shown












Required, but never shown







Required, but never shown







Popular posts from this blog

Quarter-circle Tiles

build a pushdown automaton that recognizes the reverse language of a given pushdown automaton?

Mont Emei